Difference between revisions of "2014 AIME II Problems/Problem 10"

(Solution 3)
m (Solution 2 (short but a little bashy))
(6 intermediate revisions by 4 users not shown)
Line 4: Line 4:
  
 
==Solution 1 (long but non-bashy)==  
 
==Solution 1 (long but non-bashy)==  
 
*Commenter's note: this solution made me lose brain cells, as it is so wordy for something so simple.
 
  
 
Note that the given equality reduces to
 
Note that the given equality reduces to
Line 35: Line 33:
 
and we are done. <math>\blacksquare</math>
 
and we are done. <math>\blacksquare</math>
  
==Solution 2 (short but a little bashy)==
+
==Solution 2 (short)==
 
Assume <math>z = 2014</math>. Then
 
Assume <math>z = 2014</math>. Then
 
<cmath>\frac{1}{2014 + w} = \frac{1}{2014} + \frac{1}{w}</cmath>
 
<cmath>\frac{1}{2014 + w} = \frac{1}{2014} + \frac{1}{w}</cmath>
Line 50: Line 48:
  
 
==Solution 3==
 
==Solution 3==
Our equation can be simplified like the following.
+
Notice that <cmath>\frac1{w+z} = \frac{w+z}{wz} \implies 0 = w^2 + wz + z^2 = \frac{w^3-z^3}{w-z}.</cmath>
<cmath>\frac{1}{w+z} = \frac{w+z}{wz}</cmath>
+
Hence, <math>w=ze^{2\pi i/3},ze^{4\pi i/3}</math>, and <math>P</math> is an equilateral triangle with circumradius <math>2014</math>. Then, <cmath>[P]=\frac{3}{2}\cdot 2014^2\cdot\sin\frac{\pi}3=3\cdot 1007^2\sqrt3,</cmath>
<cmath>wz = {(w+z)}^2</cmath>
+
and the answer is <math>3\cdot 1007^2\equiv 3\cdot 7^2\equiv\boxed{147}\pmod{1000}</math>.
<cmath>w^2 + wz + z^2 = 0</cmath>
 
We recognize this as the Law of Cosines with angle <math>120</math> degrees.
 
Our polygon is an equilateral triangle, say <math>ABC</math>, with center <math>O</math> at the origin and <math>AO=BO=CO=2014</math>. The area of <math>ABC</math> is <math>3*[ABO]=3*(1007*1007\sqrt{3})=3*1007^2*\sqrt{3}=3042147\sqrt{3}</math>. Thus, the answer is <math>\boxed{147}</math>.
 
  
-Solution by TheUltimate123
+
==Notes==
 +
This problem is killed by polar coordinates (complex numbers) . Solve using cosine-i-sine.
  
 
== See also ==
 
== See also ==
 
{{AIME box|year=2014|n=II|num-b=9|num-a=11}}
 
{{AIME box|year=2014|n=II|num-b=9|num-a=11}}
 
{{MAA Notice}}
 
{{MAA Notice}}

Revision as of 02:35, 20 February 2021

Problem

Let $z$ be a complex number with $|z|=2014$. Let $P$ be the polygon in the complex plane whose vertices are $z$ and every $w$ such that $\frac{1}{z+w}=\frac{1}{z}+\frac{1}{w}$. Then the area enclosed by $P$ can be written in the form $n\sqrt{3}$, where $n$ is an integer. Find the remainder when $n$ is divided by $1000$.

Solution 1 (long but non-bashy)

Note that the given equality reduces to

\[\frac{1}{w+z} = \frac{w+z}{wz}\] \[wz = {(w+z)}^2\] \[w^2 + wz + z^2 = 0\] \[\frac{w^3 - z^3}{w-z} = 0\] \[w^3 = z^3, w \neq z\]

Now, let $w = r_w e^{i \theta_w}$ and likewise for $z$. Consider circle $O$ with the origin as the center and radius 2014 on the complex plane. It is clear that $z$ must be one of the points on this circle, as $|z| = 2014$.

By DeMoivre's Theorem, the complex modulus of $w$ is cubed when $w$ is cubed. Thus $w$ must lie on $O$, since its the cube of its modulus, and thus its modulus, must be equal to $z$'s modulus.

Again, by DeMoivre's Theorem, $\theta_w$ is tripled when $w$ is cubed and likewise for $z$. For $w$, $z$, and the origin to lie on the same line, $3 \theta_w$ must be some multiple of 360 degrees apart from $3 \theta_z$ , so $\theta_w$ must differ from $\theta_z$ by some multiple of 120 degrees.

Now, without loss of generality, assume that $z$ is on the real axis. (The circle can be rotated to put $z$ in any other location.) Then there are precisely two possible distinct locations for $w$; one is obtained by going 120 degrees clockwise from $z$ about the circle and the other by moving the same amount counter-clockwise. Moving along the circle with any other multiple of 120 degrees in any direction will result in these three points.

Let the two possible locations for $w$ be $W_1$ and $W_2$ and the location of $z$ be point $Z$. Note that by symmetry, $W_1W_2Z$ is equilateral, say, with side length $x$. We know that the circumradius of this equilateral triangle is $2014$, so using the formula $\frac{abc}{4R} = [ABC]$ and that the area of an equilateral triangle with side length $s$ is $\frac{s^2\sqrt{3}}{4}$, so we have

\[\frac{x^3}{4R} = \frac{x^2\sqrt{3}}{4}\] \[x = R \sqrt{3}\] \[\frac{x^2\sqrt{3}}{4} = \frac{3R^2 \sqrt{3}}{4}\]

Since we're concerned with the non-radical part of this expression and $R = 2014$,

\[\frac{3R^2}{4} \equiv 3 \cdot 1007^2 \equiv 3 \cdot 7^2 \equiv \boxed{147} \pmod{1000}\]

and we are done. $\blacksquare$

Solution 2 (short)

Assume $z = 2014$. Then \[\frac{1}{2014 + w} = \frac{1}{2014} + \frac{1}{w}\]

\[2014w = w(2014 + w) + 2014(2014 + w)\]

\[2014w = 2014w + w^2 + 2014^2 + 2014w\]

\[0 = w^2 + 2014w + 2014^2\]

\[w = \frac{-2014 \pm \sqrt{2014^2 - 4(2014^2)}}{2} = -1007 \pm 1007\sqrt{3}i\]

Thus $P$ is an isosceles triangle with area $\frac{1}{2}(2014 - (-1007))(2\cdot 1007\sqrt{3}) = 3021\cdot 1007\sqrt{3}$ and $n \equiv 7\cdot 21\equiv \boxed{147} \pmod{1000}.$

Solution 3

Notice that \[\frac1{w+z} = \frac{w+z}{wz} \implies 0 = w^2 + wz + z^2 = \frac{w^3-z^3}{w-z}.\] Hence, $w=ze^{2\pi i/3},ze^{4\pi i/3}$, and $P$ is an equilateral triangle with circumradius $2014$. Then, \[[P]=\frac{3}{2}\cdot 2014^2\cdot\sin\frac{\pi}3=3\cdot 1007^2\sqrt3,\] and the answer is $3\cdot 1007^2\equiv 3\cdot 7^2\equiv\boxed{147}\pmod{1000}$.

Notes

This problem is killed by polar coordinates (complex numbers) . Solve using cosine-i-sine.

See also

2014 AIME II (ProblemsAnswer KeyResources)
Preceded by
Problem 9
Followed by
Problem 11
1 2 3 4 5 6 7 8 9 10 11 12 13 14 15
All AIME Problems and Solutions

The problems on this page are copyrighted by the Mathematical Association of America's American Mathematics Competitions. AMC logo.png